Mathematical and Physical Journal
for High Schools
Issued by the MATFUND Foundation
Already signed up?
New to KöMaL?

Problem C. 1679. (September 2021)

C. 1679. Prove that the value of the expression

\(\displaystyle 1-\frac{1}{2}+\frac{1}{3}-\frac{1}{4}+\ldots+\frac{1}{2021}-\frac{1}{2022} \)

is between \(\displaystyle 0\) and \(\displaystyle 1\).

(5 pont)

Deadline expired on October 11, 2021.


Sorry, the solution is available only in Hungarian. Google translation

1. megoldás. Legyen \(\displaystyle S:=1-\frac{1}{2}+\frac{1}{3}-\frac{1}{4}+\dots+\frac{1}{2021}-\frac{1}{2022}\).

Mivel

\(\displaystyle 1-\frac12,\quad \frac13-\frac14, \quad \dots, \frac{1}{2021}-\frac{1}{2022}\)

mind pozitívak, ezért összegük, \(\displaystyle S\) is az.

Hasonlóan, mivel

\(\displaystyle \frac12-\frac13,\quad \frac14-\frac15,\quad \dots, \frac{1}{2020}-\frac{1}{2021}\)

mind pozitívak, ezért összegük, \(\displaystyle 1-S-\frac{1}{2022}\) is az. Ez azt jelenti, hogy

\(\displaystyle S<1-\frac{1}{2022}<1.\)

Tehát \(\displaystyle S\) értéke valóban 0 és 1 közötti.

2. megoldás. A megoldás során az

\(\displaystyle \frac{1}{a}-\frac{1}{a+1}=\frac{1}{a(a+1)}\)

összefüggést fogjuk többször is alkalmazni.

Először is,

\(\displaystyle S=\left(1-\frac{1}{2}\right)+\left(\frac{1}{3}-\frac{1}{4}\right)+\dots+\left(\frac{1}{2021}-\frac{1}{2022}\right)=\frac{1}{1\cdot 2}+\frac{1}{3\cdot 4}+\dots+\frac{1}{2021\cdot 2022},\)

amiből egyből látható, hogy \(\displaystyle S>0\), hiszen 1011 pozitív tag összegeként állítottuk elő.

Ha \(\displaystyle S\)-hez hozzáadjuk a

\(\displaystyle T:=\frac{1}{2\cdot 3}+\frac{1}{4\cdot 5}+\dots+\frac{1}{2020\cdot 2021}\)

pozitív összeget, majd a fenti átalakítást visszafelé is elvégezzük, egy teleszkopikus összeget kapunk:

\(\displaystyle S+T=\frac{1}{1\cdot 2}+\frac{1}{2\cdot 3}+\frac{1}{3\cdot 4} +\dots+\frac{1}{2021\cdot 2022}=\)

\(\displaystyle =\left(1-\frac{1}{2}\right)+\left(\frac{1}{2}-\frac{1}{3}\right)+\left(\frac{1}{3}-\frac{1}{4}\right)+\dots+\left(\frac{1}{2020}-\frac{1}{2021}\right)+\left(\frac{1}{2021}-\frac{1}{2022}\right)=1-\frac{1}{2022},\)

amiből \(\displaystyle T>0\) alapján

\(\displaystyle S<1-\frac{1}{2022}<1.\)

Tehát \(\displaystyle S\) értéke valóban 0 és 1 közötti.


Statistics:

257 students sent a solution.
5 points:98 students.
4 points:36 students.
3 points:39 students.
2 points:28 students.
1 point:19 students.
0 point:7 students.
Unfair, not evaluated:2 solutionss.
Not shown because of missing birth date or parental permission:4 solutions.

Problems in Mathematics of KöMaL, September 2021